PT8.S4.Q14 - The dean of computing

aj412garciaaj412garcia Member
edited March 2021 in Logical Reasoning 55 karma

My brain melted trying to understand AC A..

= (

Comments

  • 1050 karma

    Hey! I'd be happy to try and explain. Are you having trouble with the stimulus, answer choice A, or both?

  • SSBM1000SSBM1000 Member
    edited March 2021 614 karma

    Hello,

    A is correct because it shows that the stimulus' conclusion is too narrow. The stimulus lays out multiple requirements for becoming the dean of computing, and then concludes that they have to pick a professor from the computer science department, thus making the unstated assumption that the only candidates who fit all the requirements are professors from the computer science department. Answer choice A weakens this argument by essentially stating that there are candidates who meet all the requirements listed in the stimulus but who are not professors, thus weakening the argument by showing that the argument's conclusion is too narrow since there are more eligible candidates other than professors from the computer science department.

    This is definitely a tricky question, so I'd be happy to clarify my explanation or answer any questions you might have about this problem.

Sign In or Register to comment.